Aref

New Member
ارسال ها
1,262
لایک ها
1,008
امتیاز
0
#1
سلام.
قصد دارم یه ماراتن راه بندازم که سطحش مرحله دو و بالاتره.
قوانین ماراتن:
1-هرکسی سوال حل کرد باید سوال بزاره
2-شماره دار باشه سوالا.
3-از این استفاده کنید.

پست های بی ارتباط به تاپیک حذف خواهد شد. به قوانین سایت هم توجه فرمایید. اگر سوالی بود پیغام شخصی بدید.


سوال 1.

تمام زوج های
از اعداد صحیح نامنفی را بیابید که
.
 

hkh74

New Member
ارسال ها
213
لایک ها
392
امتیاز
0
#2
پاسخ : ماراتن مسائل ممتاز

اگر m,n که n>1 جواب مسئله باشه دلتای معادله(ی درجه 2 بر حسب m) صحیح (و فرد) میشه پس
به وضوح یا n,t هر دو زوجند یا هر دو فرد.
1) n,t زوج:
تناقض!

2) n,t فرد:
که برای n<11 برقراره.

پس در کل n<11 و جواب های زیر به دست میان:



سوال 2.

دایره محاطی خارجی رأس
از مثلث
بر اضلاع
به ترتیب در
مماس است. اگر وسط
روی دایره ی محیطی مثلث باشد ثابت کنید وسط
هم روی دایره ی محیطی است.
 

Aref

New Member
ارسال ها
1,262
لایک ها
1,008
امتیاز
0
#3
پاسخ : ماراتن مسائل ممتاز

مرسی، درسته، فقط یه نکته ای این که به دلتا نیازی نبود.
 

zz_torna2

New Member
ارسال ها
300
لایک ها
254
امتیاز
0
#4
پاسخ : ماراتن مسائل ممتاز

سوال 2.

دایره محاطی خارجی رأس
از مثلث
بر اضلاع
به ترتیب در
مماس است. اگر وسط
روی دایره ی محیطی مثلث باشد ثابت کنید وسط
هم روی دایره ی محیطی است.
راهنمایی:ثابت کنید قوت مرکز دایره محاطی خارجی نسبت به دایره محیطی برابر مربع شعاعش هست بعد ......:3:

سوال3 .
فرض کنید a,b,c,d,e ,f اعداد طبیعی اند.و S=a+b+c+d+e+f میشمارد abc+def و ab+bc+ca-de-ef-fd را.ثابت کنید S مرکب است.
 

mntjmath

New Member
ارسال ها
36
لایک ها
44
امتیاز
0
#5
پاسخ : ماراتن مسائل ممتاز

فرض كنيد
و
به ترتيب وسط
و
باشند.با زاويه بازي خواهيم داشت
از طرفي
پس
روي دايره محيطي

است كه همان
است پس حكم ثابت شد.

سوال 4.
اعدادي حقيقي و نامنفي هستند.
و


ثابت كنيد
 
آخرین ویرایش توسط مدیر

mntjmath

New Member
ارسال ها
36
لایک ها
44
امتیاز
0
#6
پاسخ : ماراتن مسائل ممتاز

راهنمايي : چند جمله اي مقابل را در نظر بگيريد


سوال 4 رو تو يه پست ديگه نوشتم.
 

hkh74

New Member
ارسال ها
213
لایک ها
392
امتیاز
0
#7
پاسخ : ماراتن مسائل ممتاز

سوال 2.

دایره محاطی خارجی رأس
از مثلث
بر اضلاع
به ترتیب در
مماس است. اگر وسط
روی دایره ی محیطی مثلث باشد ثابت کنید وسط
هم روی دایره ی محیطی است.
راه خودم:
وسط
روی دایره ی محیطی مثلثه یعنی عمود منصف
یعنی تجانس
خط
به مرکز
(که همان محور اصلی نقطه ی
و دایره ی محاطی خارجی است) از
می گذرد.
روی محور اصلی دو نقطه ی
هم هست پس باید روی محور اصلی نقطه ی
و دایره ی محاطی خارجی باشه پس تجانس
خط
به مرکز
(=عمودمنصف
) از
می گذرد پس
روی دایره ی محیطیه.

سوال 4.
اعدادي حقيقي و نامنفي هستند.
و


ثابت كنيد
یه راه حل زشت! : (ویرایش: البته خیلی هم زشت نیست!!!
گفتم شاید سوال راه قشنگی داشته باشه)
میشه فرض کرد


سوال 5.
مرکز ارتفاعی مثلث
و
پای ارتفاع های
اند. دایره های محیطی مثلث های
در نقاط
متقاطع اند. ثابت کنید نیمسازهای زوایای
روی
همرسند.
 
آخرین ویرایش توسط مدیر

zz_torna2

New Member
ارسال ها
300
لایک ها
254
امتیاز
0
#8
پاسخ : ماراتن مسائل ممتاز

فرض كنيد
و
به ترتيب وسط
و
باشند.با زاويه بازي خواهيم داشت
از طرفي
پس
روي دايره محيطي

است كه همان
است پس حكم ثابت شد.
ببخشید مطمئنید :
درسته؟ و همچنین اگه
اونوقت AM نیمساز هستش و A,M,C و مرکز دایره محاطی خارجی هم خط میشن و مثلثی نخواهیم داشت؟! :30:
سوال 5.
مرکز ارتفاعی مثلث
و
پای ارتفاع های
اند. دایره های محیطی مثلث های
در نقاط
متقاطع اند. ثابت کنید نیمسازهای زوایای
روی
همرسند.
محل برخورد نیمساز زاویه BHC با BC را X بگیرید بعد از این قضیه که نیمساز ضلع را به نسبت دو ضلع مجاور قطع میکند و از تشابه مثلثای BEH ,CHD و FBE,FDC داریم:

پس تمامه!
 

zz_torna2

New Member
ارسال ها
300
لایک ها
254
امتیاز
0
#9
پاسخ : ماراتن مسائل ممتاز

سوال 6.
دایره محاطی داخلی مثلث abc اضلاع ac,ab را در e,f قطع میکند.نیمساز های زوایای b,c عمود منصف نیمساز al را به ترتیب در q,p قطع میکنند.ثابت کنید fp,eq,bc همرس اند.
 

mntjmath

New Member
ارسال ها
36
لایک ها
44
امتیاز
0
#10
پاسخ : ماراتن مسائل ممتاز

ببخشید مطمئنید :
درسته؟
من حواسم نبود p,q,r رو رو اضلاع متناظرشون طبق صورت مسئله نگرفته بودم.pوq روي abوac گرفته بودم.تو اين حالتم چيزي شبيه به قبليه برقراره.
 

mntjmath

New Member
ارسال ها
36
لایک ها
44
امتیاز
0
#11
پاسخ : ماراتن مسائل ممتاز

یه راه حل زشت! : (ویرایش: البته خیلی هم زشت نیست!!! :D گفتم شاید سوال راه قشنگی داشته باشه)
میشه فرض کرد

راه خودمم همينه ولي با حسابي هندسي راحت تري مي شد گفتش واسه اينكه بگي 8abc=<b+c/2 ميشد نوشت a=1-b-c بعدش يه حسابي هندسي 4تايي. 99% تشابه داره راه حلا!!
 

hkh74

New Member
ارسال ها
213
لایک ها
392
امتیاز
0
#12
پاسخ : ماراتن مسائل ممتاز

سوال 6.
دایره محاطی داخلی مثلث abc اضلاع ac,ab را در e,f قطع میکند.نیمساز های زوایای b,c عمود منصف نیمساز al را به ترتیب در q,p قطع میکنند.ثابت کنید fp,eq,bc همرس اند.
محاطیه، چون
. پس
به طور مشابه
.

از سه رابطه ی
با زاویه بازی در میاد که
برای مثلث
مرکز ارتفاعیه، یعنی اضلاع دو مثلث
و
دو به دو موازین (
محل تماس دایره محاطی با
) یا به عبارت دیگه این دو مثلث متجانس اند که همرسی مورد نظر سوال رو ثابت می کنه.


سوال 7.
تمام توابع
را بیابید که برای هر m,n صحیح داشته باشیم:

 
لایک ها Aref

Aref

New Member
ارسال ها
1,262
لایک ها
1,008
امتیاز
0
#13
پاسخ : ماراتن مسائل ممتاز

سوال 7.
تمام توابع
را بیابید که برای هر m,n صحیح داشته باشیم:

را معادله به دست آمده برای مقادیر مرتب
می گیریم.




تعریف می کنیم:

پس چون
بنابراین
.
پس اگر

در غیر این صورت عدد صحیح ناصفر
وجود دارد که
.

می خواهیم ثابت کنیم
فرض کنید این طور نباشد و عدد صحیح
یافت شود که
.

ولی چون

پس

قرار دهید



که تناقض است. پس
و در نتیجه
.
 

Aref

New Member
ارسال ها
1,262
لایک ها
1,008
امتیاز
0
#14
پاسخ : ماراتن مسائل ممتاز

سوال 8:


مجموع سری روبرو را بیابید:



PS:
اول دو تا کسر مثبتند، دو کسر بعدی منفی، دو کسر بعدی مثبت، دو کسر بعدی منفی و الی آخر.
اصلاح شد.
 
آخرین ویرایش توسط مدیر

mahanmath

New Member
ارسال ها
898
لایک ها
701
امتیاز
0
#15
پاسخ : ماراتن مسائل ممتاز

سوال 7.
تمام توابع
را بیابید که برای هر m,n صحیح داشته باشیم:

حالت اول)
فرض کنید برای عدد ناصفر
داشته باشیم
، در آن صورت با قرار دادن
در شرط مسأله نتیجه میشود
، یعنی تابع متناوب است، پس متناهی مقدار تو بردش هست، حالا قرار بدید
، در این صورت سمت راست معادله نشان میدهد 2 برابر این مقدار هم برای تابع قابل کسب هست ! پس

حالت دوم)
اگر فقط
بتواند صفر باشد با قرار دادن
میبینیم که
، از این جا جواب دوم تابع نیز بدست میآید :


سوال 9.
حداقل چند زیر مجموعه از یک مجموعه
عضوی باید انتخاب کرد که همواره در بین آن ها بتوان 3 زیرمجموعه حداقل یک عضوی پیدا کرد که 2 به 2 اشتراک داشته باشند ولی اشتراک هر 3 آن ها تهی باشد؟


ویرایش : به دلیل ناهماهنگی من، الان 2 تا سوال داریم، دیگه خودتون به ترتیب برید لطفا :D

عارف صورت سوال 8 چیه ؟ چه عدد هایی با علامت منفی اومدن :) ؟
 
آخرین ویرایش توسط مدیر

hkh74

New Member
ارسال ها
213
لایک ها
392
امتیاز
0
#16
پاسخ : ماراتن مسائل ممتاز

سوال 8:
مجموع سری روبرو را بیابید:

برای
سری تیلور لگاریتم به صورت مقابله:
. فرض کنید
ریشه ی هشتم واحد باشه.
برای
با یه کم محاسبات میشه دید که:

برای راحتی مجموع اول رو با
و دومی رو با
نشون میدیم!

میشه ثابت کرد
روی 1 پیوسته اند (اما الآن فرض می کنیم که پیوسته اند!) (از همگرایی سری صورت مسئله میشه پیوستگی رو نتیجه گرفت - تا انتهای راه رو بخونید بعداً مشخص میشه) پس تساوی برای x=1 هم برقراره:

(از این رابطه استفاده شده که
، , و این که
بر هم عمودن که هندسی ثابت میشه)
هم همون سری مورد نظره.
(ویرایش شد-عید همه مبارک باشه!)
 
آخرین ویرایش توسط مدیر

Aref

New Member
ارسال ها
1,262
لایک ها
1,008
امتیاز
0
#17
پاسخ : ماراتن مسائل ممتاز

برای سوال 9، عددی که من به دست آوردم
ه.:167: یه راهنمایی می کنید؟
 

Aref

New Member
ارسال ها
1,262
لایک ها
1,008
امتیاز
0
#19
پاسخ : ماراتن مسائل ممتاز

پس باید یه مشکلی باشه چون این مجموعه ها تعدادشون
تاست و این خاصیتی که گفته شده رو ندارند:
اگر مجموعه ی n عضوی اولیه
باشه:

به اضافه ی مجموعه های زیر:

حالا اگر سه تا مجموعه پیدا بشه که حداقل یک عضوی باشند، دو به دو اشتراکشون ناتهی باشه و اشترکشون تهی باشه باید همگی حداقل دو عضوی باشند و در این صورت همگی در 1 مشترک خواهند بود. پس
نباید جواب باشه.
 

mntjmath

New Member
ارسال ها
36
لایک ها
44
امتیاز
0
#20
پاسخ : ماراتن مسائل ممتاز

فكر كنم منظور ماهان زير مجموعه هاي حداقل دو عضوي بوده كه اون جوابش
هست
 
بالا